LSAT and Law School Admissions Forum

Get expert LSAT preparation and law school admissions advice from PowerScore Test Preparation.

 Administrator
PowerScore Staff
  • PowerScore Staff
  • Posts: 8919
  • Joined: Feb 02, 2011
|
#22811
Complete Question Explanation

Weaken-CE. The correct answer choice is (B)

The author here makes a fairly basic, though questionable, argument in this stimulus:

Premise: Marijuana can inactivate herpesviruses.

Premise: Inactivated herpesviruses can convert healthy cells to cancerous.

Conclusion: Marijuana can cause cancer.

The question requires that we seek the answer choice which would cause one to question the author's conclusion.

Answer choice (A): This answer choice certainly doesn't weaken the argument in the stimulus. If anything this choice strengthens the author's conclusion, lending more credibility to the cited results.

Answer choice (B): This is the correct answer choice. If there exist other ingredients in marijuana which neutralize the harmful effects, then this of course weakens the conclusion that marijuana causes cancer.

Answer choice (C): This answer choice strengthens the assertion that marijuana use increases the risk of cancer, so this answer choice is incorrect.

Answer choice (D): The effects that might be associated with a modified THC play no role in assessing whether or not marijuana increases cancer risk.

Answer choice (E): The fact that it has been recommended for cancer patients makes it nor more or less likely that marijuana use causes cancer, so this answer choice is incorrect.
 Sdaoud17
  • Posts: 85
  • Joined: Apr 13, 2013
|
#9168
I dont feel B that strong to weaken the Argument that is why I read all the answers and found B is the best one to choose out of the 4 choices. Because B is still talking about Marijuana is one who causes cancer

Am I correct ?

Thank you
Last edited by Sdaoud17 on Mon Apr 29, 2013 6:12 pm, edited 1 time in total.
 Steve Stein
PowerScore Staff
  • PowerScore Staff
  • Posts: 1153
  • Joined: Apr 11, 2011
|
#9171
Hi,

Thanks for your message.

In that one, the author points out that THC can cause inactivated herpesviruses (let's call them IH's), and these IHs have been shown to cause cancer. The author concludes that the use of marijuana, which has the THC, causes cancer.

The question that follows is a Weaken question, so you need to find the answer choice that weakens the conclusion that marijuana causes cancer.

Answer choice B is the only choice that weakens the author's conclusion; if the cancerous effects could be neutralized by other ingredients in THC, that weakens the author's argument.

I hope that's helpful--please let me know whether that clears this one up--thanks,

~Steve
 Sdaoud17
  • Posts: 85
  • Joined: Apr 13, 2013
|
#9172
OO Now I understand thank you
 yrivers
  • Posts: 68
  • Joined: Mar 15, 2017
|
#34747
E states marijuana has been recommended for cancer patients who are free of the herpesvirus. I thought this weakened the argument because it states that it's being given to people fighting cancer (similar to B).
 Emily Haney-Caron
PowerScore Staff
  • PowerScore Staff
  • Posts: 577
  • Joined: Jan 12, 2012
|
#34773
Hi yrivers,

Just because something is being given to cancer patients doesn't mean it doesn't cause cancer. It could be that THC only causes cancer when the herpes virus is present. E says marijuana is given to cancer patients without the herpes virus, so it could be that it won't cause any problems for them even though THC causes cancer when the herpes virus is present. Does that help?
 olafimihan.k
  • Posts: 25
  • Joined: Jul 04, 2017
|
#36829
Out of the five ways to attack a causal conclusion would answer choice B be showing "that even when the effect occurs, the cause did not occur"?
 Adam Tyson
PowerScore Staff
  • PowerScore Staff
  • Posts: 5153
  • Joined: Apr 14, 2011
|
#36879
I would read answer B another way, olafimihan.k. It looks to me more like "where the cause (marijuana) is present, the effect (cancer) may still be absent.)" Your interpretation would be appropriate if the answer said something to the effect of cancer being present even when marijuana is not present. Be sure you have your purported cause and effect sorted out correctly, and then move ahead with the standard set of causal attacks (alternate cause, cause without effect, effect without cause, reversed cause and effect, data problem).

Keep up the good work!
User avatar
 jdbh2022
  • Posts: 3
  • Joined: Feb 02, 2023
|
#99463
Hi, I have a question on answer choice C.

Doesn't answer choice C also weaken the support relationship between the premise and the conclusion?

The premise states that "THC -> inactivates the herpes virus" & "Inactivated herpes virus -> turns healthy cells into cancer cells"
Therefore (conclusion) "marijuana causes cancer"

Answer choice C seems to present an alternative explanation on how marijuana may cause cancer without attacking the premise or the conclusion in the stimulus.
Answer choice C says that THC killing the virus can weaken immune system, thus rendering the body unable to fight other viruses, which might lead to cancer.
This, to me, seems like it weakens the support relationship in the stimulus between the premise and conclusion because it presents an alternative cause.

Thank you for all your help in advance!
User avatar
 Jeff Wren
PowerScore Staff
  • PowerScore Staff
  • Posts: 385
  • Joined: Oct 19, 2022
|
#99660
Hi jdbh,

When weakening an argument, you want to focus on the conclusion because what you're really trying to do is to show that the conclusion is less likely to be true. This can be done by attacking a premise, but more often it is by showing a logical gap between the premises and the conclusion.

Here, the conclusion is that "the use of marijuana can cause cancer." The problem with Answer C is that it actually supports this conclusion rather than weakening it. While it is true that it presents a possible alternate way that marijuana may cause cancer than the one described in the argument, that is still compatible with the argument and the conclusion. If anything, this answer, by providing ANOTHER way that marijuana may cause cancer, provides additional support for the conclusion and would actually strengthen the argument.

Get the most out of your LSAT Prep Plus subscription.

Analyze and track your performance with our Testing and Analytics Package.